% doumi question paper : 1page % \documentclass[a4paper, onecolumn,10pt]{article} \usepackage{tstudypaper-online-two-doumi} \settitle{?5-1 ??? ??? ??(?5-G)(2018.5.18)} \begin{document} \begin{tst}{15mm} \begin{dummyQA} \begin{question}{1}{89567}{x} 다음을 계산하시오. \titledbox{\centering $\dfrac {13} {27}+\dfrac {7} {9}$} \point{null}{7} \options {$1 \dfrac {1} {3}$} {$1 \dfrac {8} {27}$} {\userCorrect $1 \dfrac {7} {27}$} {$1 \dfrac {2} {9}$} {$1 \dfrac {10} {27}$} \endoptions \end{question} \end{dummyQA} \begin{solution} $\dfrac {13} {27}+\dfrac {7} {9}$ $=\dfrac {13} {27}+\dfrac {21} {27} = \dfrac {34} {27}$ $=1 \dfrac {7} {27}$ \end{solution} \begin{clue} \end{clue} \end{tst} \begin{tst}{15mm} \begin{dummyQA} \begin{question}{1}{89582}{x} 분수의 합이 $1$ 보다 큰 것을 찾으시오.\\ \genth[pnum]{1} $\dfrac {1} {4} + \dfrac {3} {10}$\\ \genth[pnum]{2} $\dfrac {3} {5} + \dfrac {5} {7}$ \\ (3) $\dfrac {3} {8} + \dfrac {5} {12}$\\ \point{null}{7} \options {(1)} {\userCorrect (2)} {(3)} {(1), (2)} {(2), (3)} \endoptions \end{question} \end{dummyQA} \begin{solution} \genth[pnum]{1}$\dfrac {1} {4} + \dfrac {3} {10} = \dfrac {5} {20} + \dfrac {6} {20} = \dfrac {11} {20}$ ,\\ \genth[pnum]{2}$\dfrac {3} {5} + \dfrac {5} {7} = \dfrac {21} {35} + \dfrac {25} {35} = \dfrac {46} {35} = 1 \dfrac {11} {35}$ ,\\ \genth[pnum]{3}$\dfrac {3} {8} + \dfrac {5} {12} = \dfrac {9} {24} + \dfrac {10} {24} = \dfrac {19} {24}$\\ 따라서, \genth[pnum]{2} 입니다. \end{solution} \begin{clue} \end{clue} \end{tst} \begin{tst}{15mm} \begin{dummyQA} \begin{question}{1}{89475}{x} 다음을 계산하시오. \titledbox{\centering$4\dfrac{2}{7}+3\dfrac{1}{2}$} \point{null}{7} \options {$7 \dfrac {5} {7}$} {\userCorrect $7 \dfrac {11} {14}$} {$7 \dfrac {6} {7}$} {$8 \dfrac {11} {14}$} {$8 \dfrac {6} {7}$} \endoptions \end{question} \end{dummyQA} \begin{solution} $4 \dfrac {2} {7}+3 \dfrac {1} {2}=4 \dfrac {4} {14}+3 \dfrac {7} {14}=(4+3)+( \dfrac {4} {14}+\dfrac {7} {14} )=7+\dfrac {11} {14}=7 \dfrac {11} {14}$ \end{solution} \begin{clue} \end{clue} \end{tst} \begin{tst}{15mm} \begin{dummyQA} \begin{question}{1}{89527}{x} 다음 중 분수의 합이 $1$ 보다 큰 식은 어느 것입니까? \point{null}{7} \options {$\dfrac {1} {4} + \dfrac {3} {5}$} {$\dfrac {7} {9} + \dfrac {1} {18}$} {\userCorrect $\dfrac {1} {3} + \dfrac {5} {7}$} {$\dfrac {5} {8} + \dfrac {3} {10}$} {$\dfrac {1} {4} + \dfrac {5} {7}$} \endoptions \end{question} \end{dummyQA} \begin{solution} \nth{1} $\dfrac {1} {4} + \dfrac {3} {5} = \dfrac {5} {20} + \dfrac {12} {20} = \dfrac {17} {20}$\\ \nth{2} $\dfrac {7} {9} + \dfrac {1} {18} = \dfrac {14} {18} + \dfrac {1} {18} = \dfrac {15} {18} = \dfrac {5} {6}$\\ \nth{3} $\dfrac {1} {3} + \dfrac {5} {7}$ = $\dfrac {7} {21} + \dfrac {15} {21} = \dfrac {22} {21} = 1 \dfrac {1} {21}$\\ \nth{4} $\dfrac {5} {8} + \dfrac {3} {10} = \dfrac {25} {40} + \dfrac {12} {40} = \dfrac {37} {40}$\\ \nth{5} $\dfrac {1} {4} + \dfrac {5} {7} = \dfrac {7} {28} + \dfrac {20} {28} = \dfrac {27} {28}$ \end{solution} \begin{clue} \end{clue} \end{tst} \begin{tst}{15mm} \begin{dummyQA} \begin{question}{1}{89343}{x} 다음을 계산하시오. \titledbox{$6 \dfrac {3} {4} -2 \dfrac {7} {8}$} \point{null}{7} \options {$2 \dfrac {7} {8}$} {$3 \dfrac {1} {8}$} {$3 \dfrac {3} {8}$} {$3 \dfrac {5} {8}$} {\userCorrect $3 \dfrac {7} {8}$} \endoptions \end{question} \end{dummyQA} \begin{solution} $6 \dfrac {3} {4} -2 \dfrac {7} {8} = 6 \dfrac {3 \times 2} {4 \times 2} -2 \dfrac {7} {8}$ $= 6 \dfrac {6} {8} -2 \dfrac {7} {8}$ $= 5 \dfrac {14} {8} -2 \dfrac {7} {8}$ $= ( 5-2 ) + \left( \dfrac {14} {8} - \dfrac {7} {8} \right)$ $= 3+ \dfrac {7} {8} = 3 \dfrac {7} {8}$ \end{solution} \begin{clue} \end{clue} \end{tst} \begin{tst}{15mm} \begin{dummyQA} \begin{question}{1}{89270}{x} 다음을 계산하시오. \titledbox{\centering $\dfrac {2} {3} + \dfrac {1} {2} + \dfrac {1} {6}$} \point{null}{7} \options {$\dfrac {1} {6}$} {$\dfrac {1} {4}$} {$\dfrac {1} {3}$} {$\dfrac {2} {3}$} {\userCorrect $1 \dfrac {1} {3}$} \endoptions \end{question} \end{dummyQA} \begin{solution} 앞에서부터 두 분수씩 차례로 통분하여 더합니다. \\ $\dfrac {2} {3} + \dfrac {1} {2} + \dfrac {1} {6} = \left( \dfrac {4} {6} + \dfrac {3} {6} \right) + \dfrac {1} {6} = \dfrac {7} {6} + \dfrac {1} {6}$ \\ $= \dfrac {8} {6} = 1 \dfrac {2} {6} = 1 \dfrac {1} {3}$ \end{solution} \begin{clue} \end{clue} \end{tst} \begin{tst}{15mm} \begin{dummyQA} \begin{question}{1}{89507}{x} 승준이는 탁구를 아침에 $2 \dfrac {2} {5}$ 시간 동안 쳤고, 저녁에 $1 \dfrac {2} {7}$ 시간 동안 쳤습니다. 승준이가 오늘 하루 탁구를 친 시간은 얼마입니까? \point{null}{6} \options {$2 \dfrac {34} {35}$ 시간} {$3 \dfrac {11} {35}$ 시간} {\userCorrect $3 \dfrac {24} {35}$ 시간} {$3 \dfrac {29} {35}$ 시간} {$3 \dfrac {34} {35}$ 시간} \endoptions \end{question} \end{dummyQA} \begin{solution} (오늘 하루 탁구를 친 시간)$=$ (아침에 친 시간)$+$ (저녁에 친 시간)$= 2 \dfrac {2} {5} +1 \dfrac {2} {7} = 2 \dfrac {14} {35} +1 \dfrac {10} {35}=3 \dfrac {24} {35}$ (시간) \end{solution} \begin{clue} \end{clue} \end{tst} \begin{tst}{15mm} \begin{dummyQA} \begin{question}{3}{89370}{x} 다음을 계산하여 기약분수로 나타낼 때, 분자와 분모의 합을 하시오. \titledbox{$\dfrac {9} {11}-\dfrac {3} {5}$} \point{null}{6} \end{question} \begin{correctlist} \response \tstpre{}\tstacon{}\tstpost{} \endresponse \correct \tstpre{}\tstacon{$67$}\tstpost{} \endcorrect \end{correctlist} \end{dummyQA} \begin{solution} $\dfrac {9} {11}-\dfrac {3} {5}=\dfrac {45} {55}-\dfrac {33} {55}=\dfrac {12} {55}$,\\ $12+55=67$ \end{solution} \begin{clue} \end{clue} \end{tst} \begin{tst}{15mm} \begin{dummyQA} \begin{question}{1}{89375}{x} 빈 칸에 알맞은 수를 차례대로 써넣으시오. \begin{tsttabular}{|c|c|c|c|c|} \hline $\dfrac{2}{5}$&$+\dfrac{1}{2}$&$\ \ \ $&$-\dfrac{3}{20}$&$\ \ \ $\\ \hline \end{tsttabular} \point{null}{6} \options {$\dfrac {7} {10} ,~ \dfrac {3} {4}$} {\userCorrect $\dfrac {9} {10} ,~ \dfrac {3} {4}$} {$\dfrac {7} {10} ,~ \dfrac {3} {5}$} {$\dfrac {7} {10} ,~ \dfrac {11} {20}$} {$\dfrac {9} {10} ,~ \dfrac {11} {20}$} \endoptions \end{question} \end{dummyQA} \begin{solution} $\dfrac {2} {5} + \dfrac {1} {2}= \dfrac {4} {10} + \dfrac {5} {10} = \dfrac {9} {10}$,\\ $\dfrac {9} {10} - \dfrac {3} {20} = \dfrac {18} {20} - \dfrac {3} {20} = \dfrac {15} {20} = \dfrac {3} {4}$ \end{solution} \begin{clue} \end{clue} \end{tst} \begin{tst}{15mm} \begin{dummyQA} \begin{question}{3}{89229}{x} \tstblank안에 알맞은 수를 구하시오. \centerpic{0.1}{/DBR_Repository/logoDirectory/auxiliary/2/75/132/766722329915713.png} \point{null}{5} \end{question} \begin{correctlist} \response \tstpre{}\tstacon{}\tstpost{} \endresponse \correct \tstpre{}\tstacon{$3 \dfrac {39}{40}$}\tstpost{} \endcorrect \end{correctlist} \end{dummyQA} \begin{solution} \genblank{$3\dfrac{39}{40}$}$= 12 \dfrac {1} {5} - 3 \dfrac {7} {8} - 4 \dfrac {7} {20}$ $= \left( 12 \dfrac {8} {40} - 3 \dfrac {35} {40} \right) - 4 \dfrac {7} {20}$ \\ $= 8 \dfrac {13} {40} - 4 \dfrac {7} {20} = 8 \dfrac {13} {40} - 4 \dfrac {14} {40}$ $= 7 \dfrac {53} {40} - 4 \dfrac {14} {40} = 3 \dfrac {39} {40}$ \end{solution} \begin{clue} \end{clue} \end{tst} \begin{tst}{15mm} \begin{dummyQA} \begin{question}{3}{89230}{x} \tstblank안에 알맞은 수를 써넣으시오. \centerpic{0.1}{/DBR_Repository/logoDirectory/auxiliary/2/75/135/767676235906862.png} \point{null}{5} \end{question} \begin{correctlist} \response \tstpre{}\tstacon{}\tstpost{} \endresponse \correct \tstpre{}\tstacon{$2 \dfrac {19} {40}$}\tstpost{} \endcorrect \end{correctlist} \end{dummyQA} \begin{solution} \genblank{$2 \dfrac{19}{40}$}$=11 \dfrac {1} {4}-\left(2 \dfrac {3} {8}+6 \dfrac {2} {5}\right)\\ =11 \dfrac {1} {4}- \left (2 \dfrac {15} {40}+6 \dfrac {16} {40} \right)$\\ $=11 \dfrac {1} {4}-8 \dfrac {31} {40}$\\ $=11 \dfrac {10} {40}-8 \dfrac {31} {40}=10 \dfrac {50} {40}-8 \dfrac {31} {40}$\\ $=2 \dfrac {19} {40}$ \end{solution} \begin{clue} \end{clue} \end{tst} \begin{tst}{15mm} \begin{dummyQA} \begin{question}{3}{89124}{x} \tstblank안에 알맞은 수를 써넣으시오. \titledbox{\centering $6 \dfrac {1} {4}+$\genblank{$5 \dfrac{2}{3}$}$=12 \dfrac {1} {2}-3 \dfrac {1} {6}$} \point{null}{5} \end{question} \begin{correctlist} \response \tstpre{}\tstacon{}\tstpost{} \endresponse \correct \tstpre{}\tstacon{$3 \dfrac {1} {12}$}\tstpost{} \endcorrect \end{correctlist} \end{dummyQA} \begin{solution} $12 \dfrac {1} {2}-3 \dfrac {1} {6}=12 \dfrac {3} {6}-3 \dfrac {1} {6}=9 \dfrac {1} {3}$ 이므로 \\ $6 \dfrac {1} {4}+$\genblank{$5 \dfrac{2}{3}$} $=9 \dfrac {1} {3}$ 에서 \\ \genblank{$5 \dfrac{2}{3}$}$=9 \dfrac {1} {3} -6 \dfrac {1} {4}=9 \dfrac {4} {12}-6 \dfrac {3} {12}=3 \dfrac {1} {12}$ \end{solution} \begin{clue} \end{clue} \end{tst} \begin{tst}{15mm} \begin{dummyQA} \begin{question}{3}{89190}{x} 다음 $\bigcirc$안에 $>$, $<$를 써넣으시오. \titledbox{\centering $\dfrac {5} {8}-\dfrac {7} {12}+\dfrac {1} {6}$ $\bigcirc$ $\dfrac {3} {4}-\dfrac {1} {6}-\dfrac {1} {3}$} \point{null}{6} \end{question} \begin{correctlist} \response \tstpre{}\tstacon{}\tstpost{} \endresponse \correct \tstpre{}\tstacon{$<$}\tstpost{} \endcorrect \end{correctlist} \end{dummyQA} \begin{solution} $\dfrac {5} {8}-\dfrac {7} {12}+\dfrac {1} {6}=\dfrac {15} {24} - \dfrac {14} {24} + \dfrac {4} {24}=\dfrac {5} {24}$,\\ $\dfrac {3} {4}-\dfrac {1} {6}-\dfrac {1} {3}=\dfrac {9} {12} - \dfrac {2} {12} - \dfrac {4} {12}=\dfrac {3} {12}=\dfrac {1} {4}$\\ 이므로 $\dfrac {5} {24}$ $<$ $\dfrac {1} {4}\left(=\dfrac {6} {24}\right)$ \end{solution} \begin{clue} \end{clue} \end{tst} \begin{tst}{15mm} \begin{dummyQA} \begin{question}{3}{89628}{x} 다음 분수 중 가장 큰 수와 둘째 번으로 작은 수의 합과 차를 차례대로 구하시오. \titledbox{\centering $\dfrac{4}{5}\qquad\dfrac{7}{8}\qquad\dfrac{3}{4}\qquad\dfrac{5}{6}$} \point{null}{5} \end{question} \begin{correctlist} \response \tstpre{}\tstacon{}\tstpost{} \endresponse \response \tstpre{}\tstacon{}\tstpost{} \endresponse \correct \tstpre{}\tstacon{$1\dfrac {27} {40}$}\tstpost{} \endcorrect \correct \tstpre{}\tstacon{$\dfrac {3}{40}$}\tstpost{} \endcorrect \end{correctlist} \end{dummyQA} \begin{solution} 분수의 분모를 $120$으로 통분하면 $\dfrac{96}{120}, \ \dfrac{105}{120}, \ \dfrac{90}{120}, \ \dfrac{100}{120}$\\ 큰 분수부터 써 보면\\ $\dfrac {7} {8} > \dfrac {5} {6} > \dfrac {4} {5} > \dfrac {3} {4}$ \\ 따라서 가장 큰 수는 $\dfrac{7}{8}$이고 둘째 번으로 작은 수는 $\dfrac{4}{5}$입니다.\\ 합 : $\dfrac {7} {8} + \dfrac {4} {5} = \dfrac {35} {40} + \dfrac {32} {40}=\dfrac {67} {40}=1 \dfrac {27} {40}$ \\ 차 : $\dfrac {7} {8} - \dfrac {4} {5} = \dfrac {35} {40} - \dfrac {32} {40}=\dfrac {3} {40}$ \end{solution} \begin{clue} \end{clue} \end{tst} \begin{tst}{15mm} \begin{dummyQA} \begin{question}{3}{89412}{x} 분모가 $8$ 인 대분수가 두 개 있습니다. 두 대분수의 합을 기약분수로 나타내면 $6\dfrac{1}{4}$ 이고, 차를 기약분수로 나타내면 $1\dfrac{1}{2}$ 입니다. 두 대분수 중 큰 대분수를 $\mth{1}\dfrac{\mth{2}}{8}$ 이라 할 때, $\mth{1}+\mth{2}$의 값을 구하시오. \point{null}{4} \end{question} \begin{correctlist} \response \tstpre{}\tstacon{}\tstpost{} \endresponse \correct \tstpre{}\tstacon{$10$}\tstpost{} \endcorrect \end{correctlist} \end{dummyQA} \begin{solution} 두 대분수 중에서 큰 대분수를 \genth[ogana]{1}, 작은 대분수를 \genth[ogana]{2}로 하여 그림으로 나타내면 다음과 같습니다. \charpic{0.1}{/DBR_Repository/logoDirectory/auxiliary/2/74/140/692252453519879.png} \genth[ogana]{1}와 \genth[ogana]{2}의 합이 $6 \dfrac {1} {4}$ 이고, \genth[ogana]{1}와 \genth[ogana]{2}의 차가 $1 \dfrac {1} {2}$ 이므로, \\ \genth[ogana]{1}와 \genth[ogana]{1}의 합은\\ $6 \dfrac {1} {4} + 1 \dfrac {1} {2} = 6 \dfrac {1} {4} + 1 \dfrac {2} {4} = 7 \dfrac {3} {4}$ \\ $7 \dfrac {3} {4}$을 분모가 $8$인 분수로 만들면 $7 \dfrac {6} {8}$이고, \\ $7 \dfrac {6} {8}=\dfrac {62} {8}$ 이므로, \genth[ogana]{1}는 $\dfrac {31} {8} = 3 \dfrac {7} {8}$ 입니다.\\ 따라서, 큰 대분수는 $3 \dfrac {7} {8}$ 이고, \genth[ogana]{1}=$3$ , \genth[ogana]{2}=$7$ 이 되므로, \genth[ogana]{1}+\genth[ogana]{2}=$3+7=10$이 됩니다.\\ \end{solution} \begin{solution} 두 대분수 중에서 큰 대분수를 \genth[ogana]{1}, 작은 대분수를 \genth[ogana]{2}라 하면 \genth[ogana]{1}+\genth[ogana]{2}=$6 \dfrac {1} {4}$, \genth[ogana]{1}-\genth[ogana]{2}=$1 \dfrac {1} {2}$ 입니다.\\ \genth[ogana]{1}+\genth[ogana]{2}+\genth[ogana]{1}-\genth[ogana]{2}=$6 \dfrac {1} {4}+1 \dfrac {1} {2}$ 이므로, \genth[ogana]{1}+\genth[ogana]{2}=$6 \dfrac {1} {4}+1 \dfrac {1} {2}$ 입니다.\\ \genth[ogana]{1}+\genth[ogana]{1}=$6 \dfrac {1} {4}+1 \dfrac {1} {2}=6 \dfrac {1} {4}+1 \dfrac {2} {4}=7 \dfrac {3} {4}=7 \dfrac {6} {8}=\dfrac {62} {8}$ 이므로, \genth[ogana]{1}는 $\dfrac {31} {8}=3 \dfrac {7} {8}$ 입니다.\\ 따라서, 큰 대분수는 $3 \dfrac {7} {8}$ 입니다. \end{solution} \begin{clue} \end{clue} \end{tst} \begin{tst}{15mm} \begin{dummyQA} \begin{question}{3}{89439}{x} 통에 물을 가득 채우면 그 무게가 $15 \dfrac {1} {2} \rm kg$ 이라고 합니다. 통에 가득찬 물의 $\dfrac {1} {2}$ 을 쏟고 무게를 재었더니 $8 \dfrac {3} {5} \rm kg$ 이었습니다. 통의 무게는 몇 $\rm kg$입니까? \point{null}{4} \end{question} \begin{correctlist} \response \tstpre{}\tstacon{}\tstpost{\DDkg} \endresponse \correct \tstpre{}\tstacon{$1 \dfrac {7} {10}$}\tstpost{\DDkg} \endcorrect \end{correctlist} \end{dummyQA} \begin{solution} 물의 $\dfrac {1} {2}$ 을 쏟고 무게를 재었을 때 물통과 물 무게의 $\dfrac {1} {2}$ 이 $8 \dfrac {3} {5} \rm kg$ 이므로\\ 물 무게의 $\dfrac {1} {2}$ : $15 \dfrac {1} {2} - 8 \dfrac {3} {5} = 15 \dfrac {5} {10} - 8 \dfrac {6} {10}$ $= 14 \dfrac {15} {10} - 8 \dfrac {6} {10}$ $= 6 \dfrac {9} {10} ( \rm kg)$\\ 통의 무게 : $8 \dfrac {3} {5} - 6 \dfrac {9} {10} = 7 \dfrac {16} {10} - 6 \dfrac {9} {10}$ $= 1 \dfrac {7} {10} ( \rm kg)$ \end{solution} \begin{clue} \end{clue} \end{tst} \begin{tst}{15mm} \begin{dummyQA} \begin{question}{3}{89168}{x} 어떤 수에 $2 \dfrac {1} {4}$ 을 더해야 할 것을 잘못하여 빼었더니 $7 \dfrac {5} {6}$ 가 되었습니다. 바르게 계산한 답과 잘못 계산한 답의 차를 구하시오. \point{null}{4} \end{question} \begin{correctlist} \response \tstpre{}\tstacon{}\tstpost{} \endresponse \correct \tstpre{}\tstacon{$4 \dfrac {1} {2}$}\tstpost{} \endcorrect \end{correctlist} \end{dummyQA} \begin{solution} 어떤 수를 \genblank{$1 \dfrac{13}{18}$}라 하면 \genblank{$1 \dfrac{13}{18}$}$-2 \dfrac {1} {4} = 7 \dfrac {5} {6}$, \\ \genblank{$1 \dfrac{13}{18}$}$= 7 \dfrac {5} {6} +2 \dfrac {1} {4} = 7 \dfrac {10} {12} +2 \dfrac {3} {12} = 9 \dfrac {13} {12}$ $= 10 \dfrac {1} {12}$입니다.\\ 바르게 계산하면 \\ $10 \dfrac {1} {12} +2 \dfrac {1} {4} = 10 \dfrac {1} {12} +2 \dfrac {3} {12} $\\ $= 12 \dfrac {4} {12} = 12 \dfrac {1} {3}$ 입니다.\\ $\to$ $12 \dfrac {1} {3} -7 \dfrac {5} {6} = 11 \dfrac {8} {6} -7 \dfrac {5} {6} = 4 \dfrac {3} {6} = 4 \dfrac {1} {2}$ \end{solution} \begin{clue} \end{clue} \end{tst} \begin{tst}{15mm} \begin{dummyQA} \begin{question}{3}{89705}{x} 물이 가득 들어 있는 병의 무게가 $3 \dfrac {5} {6} \DDkg$ 입니다. 규형이가 전체 물의 반을 마셨더니 물이 든 병의 무게는 $2 \dfrac {1} {3} \DDkg$ 이 되었습니다. 빈 물통만의 무게를 분수로 나타내시오. \point{null}{4} \end{question} \begin{correctlist} \response \tstpre{}\tstacon{}\tstpost{} \endresponse \correct \tstpre{}\tstacon{$\dfrac {5} {6} \DDkg$}\tstpost{} \endcorrect \end{correctlist} \end{dummyQA} \begin{solution} 전체 물의 반 : \\ $3 \dfrac {5} {6} -2 \dfrac {1} {3} = 3 \dfrac {5} {6} -2 \dfrac {2} {6} = 1 \dfrac {3} {6}= 1 \dfrac {1} {2} ( \DDkg)$ \\ 전체 물의 양 : \\ $1 \dfrac {1} {2} +1 \dfrac {1} {2} = 2 \dfrac {2} {2} = 3( \DDkg)$ \\ 빈 물병의 무게 : \\ $3 \dfrac {5} {6} - 3 = \dfrac {5} {6} ( \DDkg)$ \end{solution} \begin{clue} \end{clue} \end{tst} \begin{tst}{15mm} \begin{dummyQA} \begin{question}{3}{89742}{x} 다음 \mth{1}, \mth{2}, \mth{3}에 알맞은 수를 차례대로 구하시오. (단, $\mth{3} \ > \ \mth{2} \ > \ \mth{1}$ ) \titledbox{$\dfrac {25} {28} = \dfrac {1} {\mth{1}} + \dfrac {1} {\mth{2}} + \dfrac {1} {\mth{3}}$} \point{null}{2} \end{question} \begin{correctlist} \response \tstpre{}\tstacon{}\tstpost{} \endresponse \response \tstpre{}\tstacon{}\tstpost{} \endresponse \response \tstpre{}\tstacon{}\tstpost{} \endresponse \correct \tstpre{}\tstacon{$2$}\tstpost{} \endcorrect \correct \tstpre{}\tstacon{$4$}\tstpost{} \endcorrect \correct \tstpre{}\tstacon{$7$}\tstpost{} \endcorrect \end{correctlist} \end{dummyQA} \begin{solution} $28$ 의 약수 : $1$ , $2$ , $4$ , $7$ , $14$ , $28$ \\ $4+7+14= 25$ \\ $\dfrac {25} {28} = \dfrac {4} {28} + \dfrac {7} {28} + \dfrac {14} {28}$ =$\dfrac {1} {7} + \dfrac {1} {4} + \dfrac {1} {2}$\\ 입니다. 따라서 \mth{1}$=2$, \mth{2}$=4$, \mth{3}$=7$ \end{solution} \begin{clue} \end{clue} \end{tst} \begin{tst}{15mm} \begin{dummyQA} \begin{question}{3}{89629}{x} 다음 숫자 카드를 한 번씩 써서 $2$개의 대분수를 만들었습니다. 두 분수의 합이 가장 크게 되는 합을 구하시오.\\ \centerpic{0.1}{/DBR_Repository/logoDirectory/auxiliary/1/78/74/6027126042045117.png} \point{null}{3} \end{question} \begin{correctlist} \response \tstpre{}\tstacon{}\tstpost{} \endresponse \correct \tstpre{}\tstacon{$18$}\tstpost{} \endcorrect \end{correctlist} \end{dummyQA} \begin{solution} 자연수 부분은 가장 큰 숫자부터 쓰고, 분수 부분은 나머지 수를 가지고 \\ 가장 큰 분수와 둘째로 큰 분수를 만들어야 합니다. \\ 가장 큰 수 $2$ 개는 $8$,\ $9$이므로 \\ 이 두 숫자를 대분수의 자연수로 만듭니다.\\ 나머지 $1$, $3$, $4$, $6$을 이용하여 두 분수의 합이 가장 크게 만들 수 있는 분수는 $\dfrac {1} {3}$과 $\dfrac {4} {6}$ 입니다. \\ 따라서 두 분수는 $8 \dfrac {1} {3}$과 $9 \dfrac {4} {6}$ \\ 또는, $9 \dfrac {1} {3}$ 과 $8 \dfrac {4} {6}$ 입니다. \\ 두 분수의 합을 구하면 $8 \dfrac {1} {3} + 9 \dfrac {4} {6} =8 \dfrac {6} {18} + 9 \dfrac {12} {18}= 18$ 입니다. \end{solution} \begin{clue} \end{clue} \end{tst} \end{document}